Difference between revisions of "2018 AMC 10A Problems/Problem 1"

m
(Solution)
Line 4: Line 4:
 
===Solution===
 
===Solution===
 
Evaluating the expressions starting with the innermost one, we get that the answer is <math>\boxed{\textbf{(B)}\ \frac{11}{7}}</math>
 
Evaluating the expressions starting with the innermost one, we get that the answer is <math>\boxed{\textbf{(B)}\ \frac{11}{7}}</math>
 +
== See Also ==
 +
 +
{{AMC10 box|year=2017|ab=A|before=First Problem|num-a=2}}
 
{{MAA Notice}}
 
{{MAA Notice}}

Revision as of 15:53, 8 February 2018

What is the value of \[\left(\left((2+1)^{-1}+1\right)^{-1}+1\right)^{-1}+1?\]$\textbf{(A) } \frac58 \qquad \textbf{(B) }\frac{11}7 \qquad \textbf{(C) } \frac85 \qquad \textbf{(D) } \frac{18}{11} \qquad \textbf{(E) } \frac{15}8$

Solution

Evaluating the expressions starting with the innermost one, we get that the answer is $\boxed{\textbf{(B)}\ \frac{11}{7}}$

See Also

2017 AMC 10A (ProblemsAnswer KeyResources)
Preceded by
First Problem
Followed by
Problem 2
1 2 3 4 5 6 7 8 9 10 11 12 13 14 15 16 17 18 19 20 21 22 23 24 25
All AMC 10 Problems and Solutions

The problems on this page are copyrighted by the Mathematical Association of America's American Mathematics Competitions. AMC logo.png